oi Samuel,
muito obrigada! vou printar e ler com cuidado! pois nao tenho o tempo
necessario pro curso todo!
Valeu mesmo!!! Posso colocar no meu blog uma versao em ingles?
abs
Valeria

On Wed, Oct 9, 2019 at 10:12 AM 'Samuel Gomes' via LOGICA-L <
logica-l@dimap.ufrn.br> wrote:

> Olá todos, olá Valeria,
>
> Bem, dado o chamamento a opinar, vou fazer alguns comentários adicionais
> sobre alguns dos posts anteriores, de maneira rápida e curta... Quando eu
> quis compilar tudo o que eu gostaria de falar sobre a Hipótese do Contínuo,
> em 2015, o que saiu foi um minicurso de três sessões, cada uma com mais de
> 60 slides ! Posso enviar o arquivo para quem me pedir off-list (e aceito
> convites para ministrar esse minicurso por aí, obviamente...).
>
> 1) O enunciado original de Cantor para CH é exatamente como Doria citou:
> não existe subconjunto da reta de tamanho intermediário entre o tamanho dos
> naturais e o tamanho da reta. Se assumimos o Axioma da Escolha (para que
> todo cardinal seja um aleph, i.e., para que toda cardinalidade seja
> bem-ordenável, digamos), aí temos o enunciado bonitinho do 2^{aleph_0} =
> aleph_1.
>
> Tem coisas muito interessantes sobre esse enunciado original de Cantor,
> "não existir tamanho intermediário". Primeiro: ele pode ser demonstrado
> usando o Axioma da Determinação ! Lembrar, porém,  que o Axioma da
> Determinação (o qual declara, informalmente, que jogos cujo alvo sejam
> subconjuntos da reta sempre vão ter estratégia vencedora para algum dos
> jogadores) é incompatível com o Axioma da Escolha.
>
> Outra coisa muito interessante: se generalizamos de uma certa forma
> natural esse enunciado para qualquer conjunto infinito, chegamos num
> enunciado que podemos considerar como sendo a "Hipótese Generalizada do
> Contínuo", mas sem referência a alephs (essencialmente: "se X tem injeção
> para um certo Y que tem injeção para Partes de X, então Y tem que ser
> equipotente ou a X ou a Partes de X, não havendo possibilidades
> intermediárias"). Pois bem, esse enunciado generalizado implica o Axioma da
> Escolha !!! Isso não é muito divulgado por aí. Fiz um trabalho com um aluno
> de iniciação e destrinchamos essa equivalência, tenho os slides do projeto
> dele, também posso enviar para quem estiver interessado.
>
> 2) Outro fato que não é muito divulgado por aí: parafraseando Paulo
> Freire, "Cantor não acordou um dia às oito horas da manhã e conjecturou a
> Hipótese do Contínuo".
>
> O que ocorre é que Cantor não conseguiu PRODUZIR, com seus métodos, nenhum
> conjunto de tamanho intermediário. Slogan: "para conjuntos que possam ser
> razoavelmente descritos, a Hipótese do Contínuo é verdadeira". O nascimento
> da Teoria dos Conjuntos pode ser traçado ao Teorema de Cantor-Bendixson, no
> qual Cantor enxergou uma recursão transfinita de comprimento maior do que
> omega brincando com pontos de acumulação de subconjuntos da reta. Pois bem,
> o Teorema de Cantor-Bendixson diz que: fechados não-enumeráveis da reta são
> necessariamente equipotentes à reta (diz um pouco mais do que isso, mas por
> hoje digamos que seja isso). A construção garante que, dado um fechado
> não-enumerável, esse fechado deve conter um subconjunto perfeito - i.e., um
> fechado sem pontos isolados. Pois bem, um lindo argumento com árvores
> binárias garante que qualquer subconjunto perfeito contém uma cópia do
> Conjunto de Cantor, e portanto tem a cardinalidade do contínuo.
>
> "Conter uma cópia do conjunto de Cantor" é uma propriedade que garante à
> uma dada classe de conjuntos um certo carimbo de "regularidade", de "bom
> comportamento".
> Essa propriedade, conter uma cópia do Conjunto de Cantor, é chamada
> "propriedade do conjunto perfeito". Por exemplo, os analíticos (= projeções
> de borelianos) possuem essa propriedade (Luzin, 1917). Notar que se uma
> certa classe de conjuntos possui a propriedade do conjunto perfeito, então
> um contra-exemplo para a Hipótese do Contínuo não vai sair dessa classe !!!
> Assim, a Hipótese do Contínuo vale para toda classe de subconjuntos com a
> tal propriedade do conjunto perfeito, como são os fechados e os
> analíticos...(no sentido de que, para os fechados por exemplo, não existem
> tamanhos intermediários)
>
> Obviamente que "ser Lebesgue mensurável" é um carimbo de bom comportamento
> possivelmente ainda maior; porém, existem projetivos (= conjuntos obtidos a
> partir de uma quantidade finita de projeções e complementos partindo de um
> boreliano) não Lebesgue mensuráveis assumindo-se V = L, o tal Axioma da
> Construtibilidade.
>
> (O Axioma da Determinação prova que todo subconjunto da reta é Lebesgue
> mensurável...)
>
> 3) Não sei exatamente o que Gödel queria provar quando fez a construção do
> modelo dos construtíveis; nós teoristas de conjuntos simplesmente pensamos
> em V = L como uma prova da *consistência* da Hipótese do Contínuo (e do
> Axioma da Escolha, de quebra...). O que eu posso dizer é que Cohen, o tal
> que inventou o forcing para mostrar a consistência da negação da Hipótese
> do Contínuo, esse sim acreditava e declarou abertamento lá no seu livro
> (procurem lá nas últimas duas páginas...) que a cardinalidade do contínuo
> deveria estar até acima de aleph_omega, que é o limite dos aleph_n.
>
> O que Cohen parece pensar - e que eu, quando estou quase acordando de
> manhã ou quase dormindo de noite, tendo a pensar do mesmo jeito - é que o
> Axioma das Partes é uma espécie de animal selvagem, e exatamente por não
> poder ser dominado o continuo (que, como disse o Miraglia, mais
> estruturalmente do que ser a cardinalidade da reta é a cardinalidade das
> partes de omega !!!) não poderia ser alcançado nem usando o Axioma da
> Substituição a partir de omega... Rodrigo Freire tem uma visão muito mais
> lúcida do que eu nesse sentido, já conversamos a respeito, hehe.
>
> 4) A coisa do aleph_2: a maioria das tentativas razoáveis de falsear a
> Hipótese do Contínuo acaba levando o contínuo para aleph_2. Para começar, o
> Proper Forcing Axiom (que é uma versão mais forte do Axioma de Martin,
> sendo que este pode ser encarado como uma espécie de versão generalizada do
> Teorema de Baire para cardinais entre aleph_0 e o continuo) manda o
> contínuo para aleph_2; isso começa a envolver grandes cardinais, porque a
> consistência do Proper Forcing Axiom é usualmente obtida a partir de
> cardinais supercompactos.
>
> (Sem citar nomes: diz a lenda que, nos anos 90, um lógico brasileiro
> declarava que, se mudarmos a lógica subjacente da Teoria dos Conjuntos -
> não sei exatamente como, essa história eu só ouvi falar mesmo -, então
> poderia ser DEMONSTRADO que o contínuo vale aleph_2 !!!)
>
> 5) Os trabalhos de Woodin nos anos 90 meio que misturam tudo o que eu
> falei nos 4 comentários anteriores. Ele escreveu dois surveys no Notices of
> American Society sobre esse trabalho, e também tem aquele famoso artigo da
> revista Quanta Magazine/Scientific American de +- 2013 ("Dispute over
> infinite divides mathematicians").
>
> Contando assim em linhas gerais: como disse Miraglia, existe essa idéia de
> que deveria haver um "novo" axioma que decidisse a questão do continuum.
> Pois bem, o trabalho do Woodin nos anos 90 meio que entende esse axioma
> como sendo PD - o Axioma da Determinação Projetiva, o qual declara que os
> projetivos são determinados !!! Lembrar que AD, no qual todo subconjunto
> seria determinado, é incompatível com o Axioma da Escolha... "Não quero que
> todos subconjuntos da reta sejam determinados, mas quero que os projetivos
> sejam !!!"
>
> Não existe uma implicação direta, mas a maquinaria de grandes cardinais e
> axiomas de forcing que Woodin construiu para obter a consistência de PD
> acaba caindo em versões ainda mais fortes do Axioma de Martin (MM -
> Martin's Maximum), e, de modo similar ao Axioma de Forcing Próprio, esses
> axiomas de forcing acabam levando o contínuo para aleph_2.
>
> O que é curioso é que, depois de todo esse trabalho de mais ou menos 25
> anos que apontava para a negação da Hipótese do Contínuo, o que Woodin vem
> fazendo nos últimos dez anos vai no caminho oposto: ele está em busca de um
> certo "V = Ultimate L", que seria uma espécie de modelo padrão para a
> Teoria dos Conjuntos que teria algumas similaridades com V = L, porém seria
> compatível com "grandes grandes cardinais", o que não ocorre com V = L; "Se
> V = L, não existem mensuráveis" (Dana Scott). Isso de grandes grandes
> cardinais não é erro de digitação: os grandes grandes cardinais são aqueles
> que são definidos em termos de imersões elementares não-triviais do
> universo em modelos internos, e sabe-se que eles são "maiores ou iguais"
> (em termos de consistência) aos mensuráveis. Um cardinal mensurável é,
> tipicamente, um ponto crítico de uma imersão elementar não-trivial (= o
> menor ordinal que é movido pela imersão).
>
>
> .... Resumo da ópera:
>
> 1) A Hipótese do Contínuo vale para conjuntos que possam ser razoavelmente
> descritos;
>
> 2) A Hipótese Generalizada do Contínuo, que vale no modelo construtível e
> portanto é consistente, é mais uma afirmação nesse sentido: se tudo
> fosse muito organizado, "construtível", até a Hipótese Generalizada do
> Contínuo seria verdadeira;
>
> 3) Porém, da mesma forma que o Axioma da Escolha faz com que apareçam
> monstrinhos que não são mensuráveis - e aqui a organização vai no sentido
> contrário, de construir monstros, como no Paradoxo de Banach-Tarski !!! -,
> então assumir que existam conjuntos não-construtíveis/não-organizados
> possibilita que existam contra-exemplos para a Hipótese do Contínuo; mas
> esses contra-exemplos são, em certo sentido, "conjuntos feios";
>
> 4) O Axioma da Determinação prova a Hipótese do Contínuo como Cantor a
> conjecturou - porém, AD é incompatível como Axioma da Escolha ! Porém, o
> Axioma da Determinação Projetiva parece uma opção razoável que decide o
> contínuo como sendo aleph_2 porém necessita de grandes cardinais para ter
> sua consistência com ZFC demonstrada.
>
> ... Como em toda boa aula, espero que vocês saiam do meu texto com mais
> perguntas do que tinham antes, mas para boa parte delas muito possivelmente
> eu não sei a resposta !!!!
>
> Atés,
>
> []s  Samuel
>
>
>
>
>
>
>
>
>
>
>
>
>
>
>
> On Tuesday, October 8, 2019 at 2:24:48 PM UTC-3, Rodrigo Freire wrote:
>>
>> Artigo horroroso.
>>
>>
>>
>> https://blogs.oglobo.globo.com/ciencia-matematica/post/o-que-maquina-pode-aprender.html
>>
>>
>>
>>
>> --
> Você recebeu essa mensagem porque está inscrito no grupo "LOGICA-L" dos
> Grupos do Google.
> Para cancelar inscrição nesse grupo e parar de receber e-mails dele, envie
> um e-mail para logica-l+unsubscr...@dimap.ufrn.br.
> Para ver essa discussão na Web, acesse
> https://groups.google.com/a/dimap.ufrn.br/d/msgid/logica-l/cdbbafc0-2f86-4728-b79a-6185ced8384f%40dimap.ufrn.br
> <https://groups.google.com/a/dimap.ufrn.br/d/msgid/logica-l/cdbbafc0-2f86-4728-b79a-6185ced8384f%40dimap.ufrn.br?utm_medium=email&utm_source=footer>
> .
>


-- 
Valeria de Paiva
http://vcvpaiva.github.io/
http://www.cs.bham.ac.uk/~vdp/

-- 
Você está recebendo esta mensagem porque se inscreveu no grupo "LOGICA-L" dos 
Grupos do Google.
Para cancelar inscrição nesse grupo e parar de receber e-mails dele, envie um 
e-mail para logica-l+unsubscr...@dimap.ufrn.br.
Para ver esta discussão na web, acesse 
https://groups.google.com/a/dimap.ufrn.br/d/msgid/logica-l/CAESt%3DXv5MnRKiXiiAyk%3D2fwZPp5SHPAWtmOovdiSwY2d0KVNbQ%40mail.gmail.com.

Responder a